Welche Gesetze hindern massive Teilchen daran, sich mit Lichtgeschwindigkeit fortzubewegen?

Ich und ein paar meiner Freunde sind auf eine interessante Frage gestoßen.

Jackson spricht über den Fall, in dem Photon eine Masse ungleich Null hat. Indem er der Lagrange-Funktion einen zusätzlichen Term hinzufügt, zeigt er, wie sich die Maxwell-Gleichungen und das Coulomb-Gesetz ändern, wenn wir annehmen, dass das Photon massiv ist. Die Relativitätstheorie schränkt die Masse des Photons nicht ein, und die Masse ungleich Null für das Photon ist konsistent. Wir kommen also im Grunde zu folgendem: Wenn das Coulombsche Gesetz wahr ist, dann können wir sagen, dass die Masse des Photons Null ist.

Was wir uns also gefragt haben, ist, ob es ein anderes Gesetz gibt, wenn wir davon ausgehen, dass es richtig ist, was uns zu der Tatsache führt, dass andere massive Teilchen sich mit Geschwindigkeiten fortbewegen müssen < C ? Mir fällt ein triviales Gesetz ein, aber ich suche etwas Befriedigenderes.

Ich stelle diese Frage, weil ich keinen Widerspruch sehe, wenn sich ein massives Teilchen bewegt C . Klar, wenn man es beschleunigen will C Ab einer niedrigeren Geschwindigkeit erfordert der Prozess unendlich viel Energie und Impuls, aber wir können dies leicht umgehen, indem wir annehmen, dass sie bei dieser Geschwindigkeit erzeugt werden.

Die Frage ist also im Grunde: Warum können massive Teilchen nicht angreifen? C ? Welches Gesetz, wenn es als wahr angenommen wird, schränkt die Geschwindigkeit massiver Teilchen ein < C ?

Ich hoffe, niemand antwortet und sagt, dass es "unendliche Energie / Schwung" braucht, wenn Sie sagen C im Nenner. Diese Formel gilt nur in dem Fall v < C .

Wenn Sie möchten, dass jemand Ihre Frage beantwortet, ohne auf das Problem der „unendlichen Energie“ einzugehen, denke ich nicht, dass Ihre Frage beantwortbar ist. Ich denke nicht, dass es klug ist, eine Antwort zu schließen, nur weil Sie damit unzufrieden sind. Lassen Sie mich trotzdem versuchen, darauf zu antworten. Aber ich werde das Thema „unendliche Energie“ verwenden. Vielleicht gefällt Ihnen die Antwort deshalb nicht, aber ich bitte Sie, offen zu bleiben. So machen wir Wissenschaft. Eine Antwort kann nicht weggeworfen werden, basierend auf dem, was wir wollen. :)
Was ist falsch am Argument der unendlichen Energie? Es ist ein gültiges Argument. Es sei denn, Sie denken aus irgendeinem Grund, dass unendliche Energie eine physikalisch realisierbare Sache ist?
@BioPhysicist Wie ich in einigen Kommentaren unten erwähnt habe, verstehe ich nicht, warum diese Formel für Partikel gilt, die sich mit c bewegen
Tut mir leid, dass ich normalerweise nicht in Kommentaren wühle :)

Antworten (5)

Sie scheinen die mathematischen Modelle, die wir verwenden und diese Teilchen beschreiben, gut zu verstehen, und dann sagen wir, wow, diese Modelle beschreiben die Realität perfekt, weil sie alle durch unsere Experimente gerechtfertigt sind, so dass massive Teilchen sich niemals mit Lichtgeschwindigkeit fortbewegen können Vakuum, das sehen wir aus den Experimenten, und die mathematischen Modelle zeigen, wie Sie sagen, auch einen Widerspruch, wenn wir von massiven Teilchen sprechen, die sich mit Lichtgeschwindigkeit im Vakuum bewegen. Aber das ist nicht das, wonach Sie fragen.

Dann sehen Sie Sätze, in denen wir sagen: "Sie brauchen unendliche Energie, um ein massives Teilchen im Vakuum auf Lichtgeschwindigkeit zu beschleunigen". Es ist wahr und mathematisch gerechtfertigt, aber das ist einfach nicht das, wonach Sie fragen.

Sie fragen, "welches Gesetz verhindert, dass sie bei c erstellt werden", und wonach Sie suchen, wird als Higgs-Mechanismus bezeichnet.

Der Higgs-Mechanismus ist eine Art zu sagen, dass es etwas gibt, ein Feld, das (genau wie andere Felder) den gesamten Raum durchdringt und mit bestimmten Teilchen interagiert. Dieser Mechanismus (oder eine Möglichkeit, ein anderes physikalisches Gesetz auszudrücken, nach dem Sie suchen) unterscheidet massive und masselose Teilchen und interagiert (koppelt mit) mit den ersteren, aber nicht mit den letzteren, wodurch ein Phänomen entsteht, das wir in unseren Experimenten sehen Wie ein Gesetz besagt, können sich massive Teilchen im Vakuum nicht mit Lichtgeschwindigkeit bewegen.

Das Higgs-Feld ist ein weiteres Quantenfeld, und das Higgs-Feld und das Elektronenfeld interagieren. Das bedeutet, dass Sie ein Elektron nicht einfach als Anregung des Elektronenfelds schreiben können, sondern es muss als Anregung sowohl des Elektrons als auch des Higgs-Felds zusammen geschrieben werden. Da die Wechselwirkung relativ klein ist, können wir die Anregung als leicht gestörte Elektronenfeldanregung schreiben, d. h. wir schreiben sie als Anregung des Elektronenfelds plus etwas Higgs-Feld. Wenn wir nun berechnen, wie sich diese Erregung ausbreitet, finden wir, dass sie sich mit weniger als Lichtgeschwindigkeit fortbewegt, dh die Erregung der kombinierten Felder hat eine Masse. Die Menge an Masse ist proportional zur Stärke der Wechselwirkung zwischen dem Elektron und dem Higgs-Feld.

Ist das eine gute Erklärung für den Higgs-Mechanismus?

https://en.wikipedia.org/wiki/Higgs_mechanism

Nun, wir wissen nicht genau wie, aber diese Wechselwirkung mit dem Higgs-Feld, dieser Higgs-Mechanismus bewirkt irgendwie, dass diese Teilchen (die wir nachfolgend als massiv bezeichnen) immer langsamer als die Lichtgeschwindigkeit im Vakuum sind.

Bitte beachte, dass:

  1. bei Neutrinos wissen wir immer noch nicht genau, wie (durch welchen Mechanismus) sie ihre Ruhemasse erhalten

  2. Ihre Frage ist (ich nahm an, Sie fragen nach Vakuum) nur im Vakuum wahr. Massive Partikel können sich in bestimmten Medien schneller bewegen als Licht und tun dies auch

Die Frage ist also im Grunde: Warum können massive Teilchen nicht bei c gehen? Welches Gesetz, wenn es als wahr angenommen wird, beschränkt die Geschwindigkeit eines massiven Teilchens auf <c?

Ich bin mir nicht sicher, ob dies ein „Gesetz“ in dem Sinne ist, wie Sie es meinen, es ist eigentlich nur Mathematik. In Einheiten mit c=1 haben wir die folgenden zwei Gleichungen, die immer für alle Teilchen (massive, masselose und sogar hypothetische Tachyonen) gelten:

v = P E
M 2 = E 2 P 2

Wenn wir setzen v = 1 in der ersten Gleichung erhalten wir dann sofort E = P . Das Einsetzen in die zweite ergibt M = 0 .

Obwohl die unendliche Energie zum Beschleunigen das typische Problem ist, das identifiziert wurde, ist es nicht das einzige Problem. Ein Teilchen mit M > 0 Und v = 1 ist mathematisch inkonsistent. Aber es klingt spannender, über unendliche Energie zu sprechen als über mathematische Widersprüchlichkeiten. Daher geht die „Berichterstattung“ auf den aufregenderen Grund. Aber noch einmal, das ist nicht das einzige Problem. Die mathematische Inkonsistenz ist unvermeidlich, unabhängig davon, wie es zu diesem Zustand kommen würde.

Ich verstehe nicht, warum Sie davon ausgehen, dass die erste Gleichung für massive Teilchen gilt, die sich mit Lichtgeschwindigkeit bewegen. Ja, ich stimme zu, dass eine Massenschalenbeschränkung erforderlich ist.
@Razor, warum sollte es für massive Partikel wahr sein und für Partikel, die sich bei c bewegen, aber nicht für massive Partikel, die sich bei c bewegen?
Ich kann sehen, dass es eine "Ableitung" für massive Partikel ist, die sich mit Geschwindigkeit bewegen < C . Für Photonen nehmen wir an, dass die Masse Null ist, und verwenden die Massenschale, um zu der Gleichung zu gelangen C = E / P .
@Razor, wie Sie sagen, ist klar, dass die Gleichung für massive Partikel gilt. Es ist auch klar, dass die Gleichung für Teilchen bei c gilt. Wenn also massive Teilchen bei c möglich wären, dann müsste diese Gleichung gelten. Jegliche mathematische Inkonsistenzen, die sich aus der Kombination ergeben, sind Hinweise darauf, dass die Kombination mathematisch inkonsistent ist.
Ich sage, dass seine Gültigkeit nicht mit der Masse, sondern mit der Geschwindigkeit zusammenhängt. Die Herleitung dieser Gleichung gilt eigentlich nur für massive Teilchen mit Geschwindigkeit < C . Für Photonen können Sie davon ausgehen, dass die Masse Null ist
@Razor sagte: „Die Ableitung dieser Gleichung gilt eigentlich nur für massive Teilchen mit einer Geschwindigkeit <c“. Das ist in Ordnung, dann zeigt es den mathematischen Widerspruch in dieser Ableitung. Es spielt keine Rolle, wo Sie sich entscheiden, es zu zeigen. Ich habe später gewählt, aber in der Ableitung dieser Gleichung ist in Ordnung. In jedem Fall zeigen Sie am Ende, dass die Kombination inkonsistent ist. Entweder ist es alleine in Ordnung, die Kombination ist ungültig, QED

Einsteins Gesetz E = M 0 C 2 / 1 v 2 C 2 sagt nur, was die Energie eines massiven Teilchens bei einer bestimmten Geschwindigkeit ist v . Wo M 0 ist die Ruhemasse und C ist die Lichtgeschwindigkeit im Vakuum. Das bedeutet sicherlich nicht, dass sich ein massives Teilchen nicht mit Geschwindigkeit bewegen kann C . Es sagt einfach, dass die Energie eines so massiven Teilchens unendlich ist. Daher kann ich Ihnen zustimmen, dass die spezielle Relativitätstheorie nicht verhindert, dass ein massives Teilchen Geschwindigkeit hat C . Das Gesetz, das dies jedoch verhindert, ist das gute alte Energieeinspargesetz. Betrachten Sie einen inelastischen Stoßprozess, A + B M + N . Nehmen wir Teilchen an A , B , N sind von endlicher Masse und bewegen sich mit Geschwindigkeit v < C . Aber das Teilchen M hat endliche Masse und bewegt sich mit Geschwindigkeit C . Das Energieerhaltungsgesetz sagt also:

M A C 2 + M B C 2 = M M C 2 + M N C 2 .

Dies impliziert,

M M C 2 = M A C 2 + M B C 2 M N C 2 .

Aber nach Einsteins Massen-Energie-Äquivalenzgesetz haben wir die Energie des Teilchens M gegeben von, E = M M C 2 = . Somit ist die linke Seite der letzten Gleichung unendlich, während die rechte Seite endlich sein muss. Dies kann sicherlich nicht sein und verstößt damit gegen den Masse-Energie-Erhaltungssatz. Wenn Sie also nicht mit einem Teilchen unendlicher Energie beginnen, können Sie die Gleichung nicht ausgleichen und können daher anschließend keine Teilchen mit Ihrer gewünschten Spezifikation erzeugen.

Ich bin mir nicht sicher, ob diese Antwort Ihr Kriterium erfüllt. Aber das ist das Beste, was ich tun konnte.

Wer auch immer gegen die Antwort gestimmt hat, können Sie bitte den Grund angeben? was ist an dieser Argumentation falsch?
Es tut mir leid, ich bin derjenige, der es abgelehnt hat. Ihre Gleichung über Teilchen m, E = M M C 2 = ist falsch.
Können Sie erklären, warum Sie denken, dass es falsch ist? Soweit ich verstehe, wenn Sie die Geschwindigkeit eines Teilchens ( v ) Zu C , dann ist seine Energie unendlich, wenn er eine endliche Masse hat. Ist das nicht aus der Gleichung ersichtlich, E = M 0 C 2 / 1 v 2 / C 2 ?
Ich glaube nicht, dass diese Gleichung für Teilchen gilt, die sich mit der Geschwindigkeit c bewegen. Ihre Gültigkeit gilt für Teilchen, die sich mit einer Geschwindigkeit <c bewegen. Ich stimme zu, dass, wenn Sie auf Geschwindigkeit c beschleunigen möchten, unendliche Energie erforderlich ist, aber mein Punkt ist, dass es mit Geschwindigkeit c erzeugt werden kann (wie Photonen).
Können Sie mir eine Referenz geben, wo bewiesen ist, dass Einsteins E = M C 2 Formel gilt für v < C nur Fall?

Wenn das Teilchen Masse hat und sich immer noch bei c bewegt, ist der Vier-Impuls-Vektor zeitähnlich und daher gibt es einen Rahmen, in dem sein Impuls Null ist. Das bedeutet, wenn dieses Teilchen aus einer Reaktion entsteht, gibt es einen Rahmen, in dem sein Impuls Null ist, sich aber mit der Geschwindigkeit c fortbewegt. Die Bewegungsrichtung kann also beliebig sein. Das scheint wirklich seltsam (obwohl ich keine Inkonsistenz sehen kann).

Das scheint eine Sackgasse zu sein. Vielleicht kann man sich also folgendes Gesetz vorstellen: Teilchen sollten bei ihrer Erzeugung eine feste Bewegungsrichtung haben (wenn alle Produkte einen festen willkürlichen Impuls haben). Dies ist ein experimentell überprüfbares Gesetz. Dies ist wahrscheinlich nur eine Wiederholung der Frage, aber dies scheint befriedigender zu sein. Das ist, was ich denke, ohne das Wissen von qft.

Wir können sehen, dass massive Teilchen haben sollten v < C aus zwei Perspektiven.

  1. Definieren wir die Geschwindigkeit als infinitesimale Positionsänderung in infinitesimaler Zeit: D X = v D T . Dann die Lorentz-Invariante D X μ D X μ = C 2 D T 2 + D X 2 = ( C 2 v 2 ) D X 2 Null ist oder nicht, je nachdem, ob v = C oder v C .

Betrachten wir nun 4-Impulse P μ . Aus physikalischen Gründen erwarten wir, dass Impulse parallel zu der infinitesimalen Positionsänderung sind P μ D X μ (oder eher P μ D τ D X μ zur richtigen Zeit τ ), also bekommen wir

P μ P μ D X μ D X μ = ( C 2 v 2 ) D X 2

Wenn wir Masse definieren M durch die Gleichung P μ P μ = C 2 M 2 , dann erfordern M > 0 setzt das voraus v < C .

  1. Lassen Sie uns die Geschwindigkeit definieren als
    v ich = E P ich
    aus P μ P μ = C 2 E 2 + P 2 = C 2 M 2 , wir bekommen E = C 4 M 2 + C 2 P 2 (Ignorieren Sie die negative Energielösung für unsere Zwecke), daher
    v = E P = P C 2 M 2 + P 2 C
    Wir sehen das M > 0 Kräfte v < C mit der Grenze M 0 nachgeben v = C .

Für kleine Impulse können wir den obigen Ausdruck taylor-erweitern, um zu finden

v = P M ( 1 P 2 2 C 2 M 2 + Ö ( P M C ) 4 )
was der Standardbeziehung zwischen Geschwindigkeit und Impuls entspricht, dh v = P M , in niedriger Energiegrenze.